Convergence in distribution of a certain kind of sequence of random variables












1












$begingroup$


Let ${X_n}$ be a sequence of independent random variables such that $P(X_n=1)=P(X_n=-1)=dfrac 12 -dfrac 1{2n^2}$ and $P(X_n=n)=P(X_n=-n)=dfrac 1{2n^2}$.



Then does the sequence $Y_n=dfrac 1{sqrt n}sum_{k=1}^n X_k$ converge in distribution ? If it does, what is the limiting distribution ?



My work: $E(X_n)=0$ and $Var(X_n)=E(X_n^2)=2-dfrac 1{n^2}$. $E(Y_n^2)=2-dfrac {sum_{k=1}^ndfrac 1{k^2}}{n}$ . So at least there is no constant $c$ such that $Y_n to c$ in $L^2$. I don't know what to do next. Please help










share|cite|improve this question









$endgroup$

















    1












    $begingroup$


    Let ${X_n}$ be a sequence of independent random variables such that $P(X_n=1)=P(X_n=-1)=dfrac 12 -dfrac 1{2n^2}$ and $P(X_n=n)=P(X_n=-n)=dfrac 1{2n^2}$.



    Then does the sequence $Y_n=dfrac 1{sqrt n}sum_{k=1}^n X_k$ converge in distribution ? If it does, what is the limiting distribution ?



    My work: $E(X_n)=0$ and $Var(X_n)=E(X_n^2)=2-dfrac 1{n^2}$. $E(Y_n^2)=2-dfrac {sum_{k=1}^ndfrac 1{k^2}}{n}$ . So at least there is no constant $c$ such that $Y_n to c$ in $L^2$. I don't know what to do next. Please help










    share|cite|improve this question









    $endgroup$















      1












      1








      1





      $begingroup$


      Let ${X_n}$ be a sequence of independent random variables such that $P(X_n=1)=P(X_n=-1)=dfrac 12 -dfrac 1{2n^2}$ and $P(X_n=n)=P(X_n=-n)=dfrac 1{2n^2}$.



      Then does the sequence $Y_n=dfrac 1{sqrt n}sum_{k=1}^n X_k$ converge in distribution ? If it does, what is the limiting distribution ?



      My work: $E(X_n)=0$ and $Var(X_n)=E(X_n^2)=2-dfrac 1{n^2}$. $E(Y_n^2)=2-dfrac {sum_{k=1}^ndfrac 1{k^2}}{n}$ . So at least there is no constant $c$ such that $Y_n to c$ in $L^2$. I don't know what to do next. Please help










      share|cite|improve this question









      $endgroup$




      Let ${X_n}$ be a sequence of independent random variables such that $P(X_n=1)=P(X_n=-1)=dfrac 12 -dfrac 1{2n^2}$ and $P(X_n=n)=P(X_n=-n)=dfrac 1{2n^2}$.



      Then does the sequence $Y_n=dfrac 1{sqrt n}sum_{k=1}^n X_k$ converge in distribution ? If it does, what is the limiting distribution ?



      My work: $E(X_n)=0$ and $Var(X_n)=E(X_n^2)=2-dfrac 1{n^2}$. $E(Y_n^2)=2-dfrac {sum_{k=1}^ndfrac 1{k^2}}{n}$ . So at least there is no constant $c$ such that $Y_n to c$ in $L^2$. I don't know what to do next. Please help







      probability-theory probability-distributions random-variables






      share|cite|improve this question













      share|cite|improve this question











      share|cite|improve this question




      share|cite|improve this question










      asked Jan 12 at 11:27









      user521337user521337

      1,0501415




      1,0501415






















          1 Answer
          1






          active

          oldest

          votes


















          1












          $begingroup$

          ${Y_n}$ converges to $N(0,1)$ in distribution. To prove this let $Z_n=1$ if $X_nin {1,n}$ and $Z_n=-1$ if $X_nin {-1,-n}$. It is easy to check that $Z_n$ takes the values $pm 1$ with probability $frac 1 2 $ each. Of course, $Z_n$'s are independent. Hence, by CLT, $frac 1 {sqrt n} sumlimits_{k=1}^{n} Z_k to N(0,1)$ in distribution. Now consider $frac 1 {sqrt n} sumlimits_{k=1}^{n} (X_k-Z_k)$. Observe that $sumlimits_{k=1}^{infty} P{X_k neq Z_n} <infty$. By Borel Cantelli Lemma it follows that $X_k=Z_k$ eventually with probability $1$, so $frac 1 {sqrt n} sumlimits_{k=1}^{n} (X_k-Z_k) to 0$ with probability $1$. This completes the proof.






          share|cite|improve this answer









          $endgroup$













            Your Answer





            StackExchange.ifUsing("editor", function () {
            return StackExchange.using("mathjaxEditing", function () {
            StackExchange.MarkdownEditor.creationCallbacks.add(function (editor, postfix) {
            StackExchange.mathjaxEditing.prepareWmdForMathJax(editor, postfix, [["$", "$"], ["\\(","\\)"]]);
            });
            });
            }, "mathjax-editing");

            StackExchange.ready(function() {
            var channelOptions = {
            tags: "".split(" "),
            id: "69"
            };
            initTagRenderer("".split(" "), "".split(" "), channelOptions);

            StackExchange.using("externalEditor", function() {
            // Have to fire editor after snippets, if snippets enabled
            if (StackExchange.settings.snippets.snippetsEnabled) {
            StackExchange.using("snippets", function() {
            createEditor();
            });
            }
            else {
            createEditor();
            }
            });

            function createEditor() {
            StackExchange.prepareEditor({
            heartbeatType: 'answer',
            autoActivateHeartbeat: false,
            convertImagesToLinks: true,
            noModals: true,
            showLowRepImageUploadWarning: true,
            reputationToPostImages: 10,
            bindNavPrevention: true,
            postfix: "",
            imageUploader: {
            brandingHtml: "Powered by u003ca class="icon-imgur-white" href="https://imgur.com/"u003eu003c/au003e",
            contentPolicyHtml: "User contributions licensed under u003ca href="https://creativecommons.org/licenses/by-sa/3.0/"u003ecc by-sa 3.0 with attribution requiredu003c/au003e u003ca href="https://stackoverflow.com/legal/content-policy"u003e(content policy)u003c/au003e",
            allowUrls: true
            },
            noCode: true, onDemand: true,
            discardSelector: ".discard-answer"
            ,immediatelyShowMarkdownHelp:true
            });


            }
            });














            draft saved

            draft discarded


















            StackExchange.ready(
            function () {
            StackExchange.openid.initPostLogin('.new-post-login', 'https%3a%2f%2fmath.stackexchange.com%2fquestions%2f3070807%2fconvergence-in-distribution-of-a-certain-kind-of-sequence-of-random-variables%23new-answer', 'question_page');
            }
            );

            Post as a guest















            Required, but never shown

























            1 Answer
            1






            active

            oldest

            votes








            1 Answer
            1






            active

            oldest

            votes









            active

            oldest

            votes






            active

            oldest

            votes









            1












            $begingroup$

            ${Y_n}$ converges to $N(0,1)$ in distribution. To prove this let $Z_n=1$ if $X_nin {1,n}$ and $Z_n=-1$ if $X_nin {-1,-n}$. It is easy to check that $Z_n$ takes the values $pm 1$ with probability $frac 1 2 $ each. Of course, $Z_n$'s are independent. Hence, by CLT, $frac 1 {sqrt n} sumlimits_{k=1}^{n} Z_k to N(0,1)$ in distribution. Now consider $frac 1 {sqrt n} sumlimits_{k=1}^{n} (X_k-Z_k)$. Observe that $sumlimits_{k=1}^{infty} P{X_k neq Z_n} <infty$. By Borel Cantelli Lemma it follows that $X_k=Z_k$ eventually with probability $1$, so $frac 1 {sqrt n} sumlimits_{k=1}^{n} (X_k-Z_k) to 0$ with probability $1$. This completes the proof.






            share|cite|improve this answer









            $endgroup$


















              1












              $begingroup$

              ${Y_n}$ converges to $N(0,1)$ in distribution. To prove this let $Z_n=1$ if $X_nin {1,n}$ and $Z_n=-1$ if $X_nin {-1,-n}$. It is easy to check that $Z_n$ takes the values $pm 1$ with probability $frac 1 2 $ each. Of course, $Z_n$'s are independent. Hence, by CLT, $frac 1 {sqrt n} sumlimits_{k=1}^{n} Z_k to N(0,1)$ in distribution. Now consider $frac 1 {sqrt n} sumlimits_{k=1}^{n} (X_k-Z_k)$. Observe that $sumlimits_{k=1}^{infty} P{X_k neq Z_n} <infty$. By Borel Cantelli Lemma it follows that $X_k=Z_k$ eventually with probability $1$, so $frac 1 {sqrt n} sumlimits_{k=1}^{n} (X_k-Z_k) to 0$ with probability $1$. This completes the proof.






              share|cite|improve this answer









              $endgroup$
















                1












                1








                1





                $begingroup$

                ${Y_n}$ converges to $N(0,1)$ in distribution. To prove this let $Z_n=1$ if $X_nin {1,n}$ and $Z_n=-1$ if $X_nin {-1,-n}$. It is easy to check that $Z_n$ takes the values $pm 1$ with probability $frac 1 2 $ each. Of course, $Z_n$'s are independent. Hence, by CLT, $frac 1 {sqrt n} sumlimits_{k=1}^{n} Z_k to N(0,1)$ in distribution. Now consider $frac 1 {sqrt n} sumlimits_{k=1}^{n} (X_k-Z_k)$. Observe that $sumlimits_{k=1}^{infty} P{X_k neq Z_n} <infty$. By Borel Cantelli Lemma it follows that $X_k=Z_k$ eventually with probability $1$, so $frac 1 {sqrt n} sumlimits_{k=1}^{n} (X_k-Z_k) to 0$ with probability $1$. This completes the proof.






                share|cite|improve this answer









                $endgroup$



                ${Y_n}$ converges to $N(0,1)$ in distribution. To prove this let $Z_n=1$ if $X_nin {1,n}$ and $Z_n=-1$ if $X_nin {-1,-n}$. It is easy to check that $Z_n$ takes the values $pm 1$ with probability $frac 1 2 $ each. Of course, $Z_n$'s are independent. Hence, by CLT, $frac 1 {sqrt n} sumlimits_{k=1}^{n} Z_k to N(0,1)$ in distribution. Now consider $frac 1 {sqrt n} sumlimits_{k=1}^{n} (X_k-Z_k)$. Observe that $sumlimits_{k=1}^{infty} P{X_k neq Z_n} <infty$. By Borel Cantelli Lemma it follows that $X_k=Z_k$ eventually with probability $1$, so $frac 1 {sqrt n} sumlimits_{k=1}^{n} (X_k-Z_k) to 0$ with probability $1$. This completes the proof.







                share|cite|improve this answer












                share|cite|improve this answer



                share|cite|improve this answer










                answered Jan 12 at 11:50









                Kavi Rama MurthyKavi Rama Murthy

                56k42158




                56k42158






























                    draft saved

                    draft discarded




















































                    Thanks for contributing an answer to Mathematics Stack Exchange!


                    • Please be sure to answer the question. Provide details and share your research!

                    But avoid



                    • Asking for help, clarification, or responding to other answers.

                    • Making statements based on opinion; back them up with references or personal experience.


                    Use MathJax to format equations. MathJax reference.


                    To learn more, see our tips on writing great answers.




                    draft saved


                    draft discarded














                    StackExchange.ready(
                    function () {
                    StackExchange.openid.initPostLogin('.new-post-login', 'https%3a%2f%2fmath.stackexchange.com%2fquestions%2f3070807%2fconvergence-in-distribution-of-a-certain-kind-of-sequence-of-random-variables%23new-answer', 'question_page');
                    }
                    );

                    Post as a guest















                    Required, but never shown





















































                    Required, but never shown














                    Required, but never shown












                    Required, but never shown







                    Required, but never shown

































                    Required, but never shown














                    Required, but never shown












                    Required, but never shown







                    Required, but never shown







                    Popular posts from this blog

                    Mario Kart Wii

                    What does “Dominus providebit” mean?

                    Antonio Litta Visconti Arese